여유한위상

Hwangjy9 (토론 | 기여)님의 2019년 2월 12일 (화) 11:41 판 (→‎성질)

집합 [math]\displaystyle{ X }[/math]에 대해 [math]\displaystyle{ \mathcal{T}\subset \mathcal{P}(X) }[/math]

[math]\displaystyle{ \mathcal{T}=\{\emptyset\}\cup \{O\subset X\mid X\setminus O\text{ is finite}\} }[/math]

로 정의하면 [math]\displaystyle{ (X,\mathcal{T}) }[/math]위상공간이다. 이때 [math]\displaystyle{ \mathcal{T} }[/math]여유한위상(cofinite topology, finite complement topology)이라고 한다.

증명

[math]\displaystyle{ (X,\mathcal{T}) }[/math]가 위상공간의 정의를 충족함을 보인다.

  1. 당연히 [math]\displaystyle{ \emptyset\in \mathcal{T} }[/math]이고, [math]\displaystyle{ X\setminus X=\emptyset }[/math]유한집합이므로 [math]\displaystyle{ X\in\mathcal{T} }[/math]이다.
  2. 임의의 [math]\displaystyle{ O_\alpha \in \mathcal{T} }[/math] (단, [math]\displaystyle{ \alpha\in I }[/math]이며 [math]\displaystyle{ I }[/math]첨자집합)에 대해, [math]\displaystyle{ X\setminus O_\alpha }[/math][math]\displaystyle{ X }[/math]이거나 유한집합이므로 드 모르간의 법칙에 의해 [math]\displaystyle{ X\setminus \bigcup_{\alpha\in I}O_\alpha=\bigcap_{\alpha\in I}(X\setminus O_\alpha) }[/math][math]\displaystyle{ X }[/math] 또는 유한집합이다. 따라서 [math]\displaystyle{ \bigcup_{\alpha\in I}O_\alpha\in\mathcal{T} }[/math]이다.
  3. 임의의 [math]\displaystyle{ O_i\in\mathcal{T} }[/math] (단, [math]\displaystyle{ i=1,\dots, n }[/math])에 대해, [math]\displaystyle{ X\setminus O_i }[/math][math]\displaystyle{ X }[/math]이거나 유한집합이므로 드 모르간의 법칙에 의해 [math]\displaystyle{ X\setminus \bigcap_{i=1}^n O_i =\bigcup_{i=1}^n (X\setminus O_i) }[/math][math]\displaystyle{ X }[/math] 또는 유한집합이다. 따라서 [math]\displaystyle{ \bigcap_{i=1}^n O_i \in\mathcal{T} }[/math]이다.

따라서 [math]\displaystyle{ (X,\mathcal{T}) }[/math]는 위상공간이다.

성질

[math]\displaystyle{ X }[/math]를 여유한위상이 부여된 위상공간이라고 하자.

  • [math]\displaystyle{ X }[/math]가 유한집합이면, 여유한위상은 이산위상이다.
  • [math]\displaystyle{ X }[/math]무한집합이면 [math]\displaystyle{ A(\subset X) }[/math]유도집합[math]\displaystyle{ A'=\begin{cases}\emptyset,&\text{$A$ is finite} \\X,&\text{$A$ is infinite} \end{cases} }[/math]이다.
    [math]\displaystyle{ A }[/math]가 무한집합인 경우: 임의의 [math]\displaystyle{ x\in X }[/math][math]\displaystyle{ x }[/math]를 포함하는 열린집합 [math]\displaystyle{ O_x }[/math]를 고르자. 그러면 [math]\displaystyle{ X\setminus O_x }[/math]는 유한집합이고 따라서 [math]\displaystyle{ X\setminus (O_x \setminus \{x\}) }[/math]도 유한집합이다. [math]\displaystyle{ X\setminus (O_x \setminus \{x\}) }[/math][math]\displaystyle{ A }[/math]의 원소를 가지지 않는다고 가정하면 [math]\displaystyle{ (O_x \setminus \{x\})\subset X\setminus A }[/math]이어서 [math]\displaystyle{ A\subset X\setminus (O_x \setminus \{x\}) }[/math]이므로 [math]\displaystyle{ A }[/math]가 유한집합이 되어 모순이 발생한다. 따라서 [math]\displaystyle{ O_x \setminus \{x\} }[/math][math]\displaystyle{ A }[/math]의 원소를 가진다. 즉, [math]\displaystyle{ x }[/math][math]\displaystyle{ A }[/math]집적점이다.
    [math]\displaystyle{ A }[/math]가 유한집합인 경우: 임의의 [math]\displaystyle{ x\in X }[/math]에 대해 [math]\displaystyle{ O=(X\setminus A)\cup \{x\} }[/math]로 정의하면 [math]\displaystyle{ O }[/math]가 열린집합임을 알 수 있다. 그러나 [math]\displaystyle{ (O\setminus \{x\})\cap A \subset (X\setminus A)\cap A=\emptyset }[/math]이므로 [math]\displaystyle{ x\not\in A' }[/math]이다.
  • [math]\displaystyle{ X }[/math]가 무한집합이면 [math]\displaystyle{ A(\subset X) }[/math]폐포[math]\displaystyle{ \overline{A}=\begin{cases}A,&\text{$A$ is finite} \\X,&\text{$A$ is infinite} \end{cases} }[/math]이다.
    [math]\displaystyle{ \overline{A}=A\cup A' }[/math]로부터 원하는 결과를 얻는다.
  • [math]\displaystyle{ X }[/math]가 무한집합이면 [math]\displaystyle{ X }[/math]분리가능 공간이다.
    [math]\displaystyle{ X }[/math]가 무한집합이므로 [math]\displaystyle{ X }[/math]의 가산무한인 부분집합이 존재하고, 그 부분집합의 폐포가 [math]\displaystyle{ X }[/math]가 된다.
  • [math]\displaystyle{ X }[/math]비가산집합이면 [math]\displaystyle{ X }[/math]제1가산공간이 아니다. 또한 [math]\displaystyle{ X }[/math]는 어떤 점에서도 가산국소기저를 가지지 않는다.
    [math]\displaystyle{ X }[/math][math]\displaystyle{ x\in X }[/math]에서 가산국소기저 [math]\displaystyle{ \mathcal{B}_x=\{B_n:n\in\mathbb{N}\} }[/math]를 가진다고 가정하자. 그러면 각 [math]\displaystyle{ n\in\mathbb{N} }[/math]에 대해 [math]\displaystyle{ X\setminus B_n }[/math]은 유한집합이고 따라서 [math]\displaystyle{ X\setminus \bigcap_{n=1}^{\infty} B_n = \bigcup_{n=1}^{\infty}(X\setminus B_n) }[/math]은 가산집합이다. [math]\displaystyle{ X }[/math]가 비가산집합이므로 [math]\displaystyle{ \bigcap_{n=1}^{\infty} B_n }[/math]은 비가산집합이고, 따라서 [math]\displaystyle{ \bigcap_{n=1}^{\infty} B_n }[/math][math]\displaystyle{ x }[/math]와 다른 원소 [math]\displaystyle{ a }[/math]를 가진다. [math]\displaystyle{ X\setminus \{a\} }[/math][math]\displaystyle{ x }[/math]를 포함하는 열린집합이지만 임의의 [math]\displaystyle{ n\in\mathbb{N} }[/math]에 대해 [math]\displaystyle{ B_n\not\subset X\setminus \{a\} }[/math]이므로, [math]\displaystyle{ \mathcal{B}_x }[/math][math]\displaystyle{ x }[/math]의 국소기저라는 가정에 모순이 발생한다.
  • [math]\displaystyle{ (a_n) }[/math][math]\displaystyle{ X }[/math] 위의 점열이라고 하자.
    • [math]\displaystyle{ (a_n) }[/math]에서 무한 번 나타나는 원소가 존재하지 않으면, [math]\displaystyle{ (a_n) }[/math]은 모든 점으로 수렴한다.
    • [math]\displaystyle{ (a_n) }[/math]에서 무한 번 나타나는 원소가 단 하나 있으면, [math]\displaystyle{ (a_n) }[/math]은 단 한 점으로 수렴한다.
    • [math]\displaystyle{ (a_n) }[/math]에서 무한 번 나타나는 원소가 둘 이상 있으면, [math]\displaystyle{ (a_n) }[/math]은 어떤 점으로도 수렴하지 않는다.
  • [math]\displaystyle{ X }[/math]의 임의의 부분공간의 부분공간 위상은 여유한위상이다.
    [math]\displaystyle{ A }[/math][math]\displaystyle{ X }[/math]의 부분공간이라 하고, [math]\displaystyle{ T_X }[/math][math]\displaystyle{ X }[/math]의 여유한위상, [math]\displaystyle{ \mathcal{T}_A }[/math][math]\displaystyle{ T_X }[/math]에 의해 결정되는 부분공간 위상, [math]\displaystyle{ \mathcal{T}_f }[/math][math]\displaystyle{ A }[/math] 위의 여유한위상이라 하자. [math]\displaystyle{ U\in T_A }[/math]를 고르면, [math]\displaystyle{ U=O\cap A }[/math][math]\displaystyle{ O\in T_X }[/math]가 존재한다. 그러면 [math]\displaystyle{ X\setminus O }[/math]는 유한집합이고, [math]\displaystyle{ A\setminus U=A\setminus (O\cap A)=A\setminus O \subset X\setminus O }[/math]이므로 [math]\displaystyle{ A\setminus U }[/math]는 유한집합이다. 즉, [math]\displaystyle{ U\in T_f }[/math]이고 따라서 [math]\displaystyle{ T_A\subset T_f }[/math]이다. 이제 [math]\displaystyle{ V\in T_f }[/math]를 고르자. 그러면 [math]\displaystyle{ V\subset A }[/math]이고 [math]\displaystyle{ A\setminus V }[/math]는 유한집합이다. [math]\displaystyle{ V=(V\cup (X\setminus A))\cap A }[/math]이고 [math]\displaystyle{ X\setminus (V\cup (X\setminus A))=(X\setminus V)\cap A=A\setminus V }[/math]이므로 [math]\displaystyle{ V\cup (X\setminus A)\in T_X }[/math]이다. 즉, [math]\displaystyle{ V\in T_A }[/math]이고 따라서 [math]\displaystyle{ T_f \subset T_A }[/math]이다. 결국 [math]\displaystyle{ T_A=T_f }[/math]임을 알 수 있다.
  • [math]\displaystyle{ X }[/math]의 임의의 부분공간은 콤팩트공간이다.
    [math]\displaystyle{ X }[/math]의 열린덮개를 [math]\displaystyle{ \mathcal{O} }[/math]라 하자. [math]\displaystyle{ \mathcal{O} }[/math]의 원소 [math]\displaystyle{ O }[/math]를 하나 고르면 [math]\displaystyle{ X\setminus O }[/math]는 유한집합이고, 따라서 [math]\displaystyle{ X\setminus O=\{x_1,\dots, x_n\} }[/math]로 쓸 수 있다. 즉, [math]\displaystyle{ O }[/math][math]\displaystyle{ x_1,\dots, x_n }[/math]을 제외한 [math]\displaystyle{ X }[/math]의 모든 원소를 포함한다. 한편 각 [math]\displaystyle{ x_i }[/math]에 대해 [math]\displaystyle{ x_i \in O_i }[/math][math]\displaystyle{ O_i\in \mathcal{O} }[/math]가 존재한다. 그러면 [math]\displaystyle{ X=O \cup \bigcup_{i=1}^n O_i }[/math]이므로 [math]\displaystyle{ \{O,O_1,\dots, O_n\} }[/math][math]\displaystyle{ X }[/math]의 유한 열린덮개임을 알 수 있다.
  • [math]\displaystyle{ X }[/math]T1 공간이다.
    [math]\displaystyle{ X }[/math]한원소집합이면 [math]\displaystyle{ X }[/math]의 서로 다른 두 원소를 선택할 수 없으므로 원하는 결론을 얻는다. [math]\displaystyle{ X }[/math]가 원소를 두 개 이상 가진다고 가정하자. 서로 다른 [math]\displaystyle{ a,b\in X }[/math]를 고르면 [math]\displaystyle{ O_a=X\setminus \{b\} }[/math][math]\displaystyle{ a }[/math]를 포함하지만 [math]\displaystyle{ b }[/math]를 포함하지 않는 열린집합이고, [math]\displaystyle{ O_b=X\setminus \{a\} }[/math][math]\displaystyle{ b }[/math]를 포함하지만 [math]\displaystyle{ a }[/math]를 포함하지 않는 열린집합이다. 따라서 [math]\displaystyle{ X }[/math]는 T1 공간이다.
  • [math]\displaystyle{ X }[/math]의 임의의 T1 위상은 여유한위상을 포함한다.
  • [math]\displaystyle{ X }[/math]T2 공간일 필요충분조건은 [math]\displaystyle{ X }[/math]가 유한집합인 것이다.
    [math]\displaystyle{ X }[/math]가 무한집합이면 모든 항이 서로 다른 [math]\displaystyle{ X }[/math] 위의 수열을 제시할 수 있고 그 수열은 모든 점으로 수렴한다. T2 공간에서 수열의 극한값은 유일하므로 [math]\displaystyle{ X }[/math]는 T2 공간이 아니다. [math]\displaystyle{ X }[/math]가 유한집합이면 [math]\displaystyle{ X }[/math] 위의 여유한위상은 이산위상과 동일하고 이산위상이 부여된 위상공간은 T2 공간이므로 [math]\displaystyle{ X }[/math]는 T2 공간이다.

같이 보기